Wie sieht eine Galilei-Transformation der Maxwell-Gleichungen aus?

In den 1860er Jahren formulierte Maxwell die sogenannte Maxwell-Gleichung und stellte fest, dass sie zu einer bemerkenswerten Schlussfolgerung führt: der Existenz elektromagnetischer Wellen, die sich mit hoher Geschwindigkeit ausbreiten c , was sich als Lichtgeschwindigkeit herausstellt, was bedeutet, dass Licht eine elektromagnetische Welle ist. Nun ist die Tatsache, dass die Maxwell-Gleichungen die Lichtgeschwindigkeit vorhersagen, eine Tatsache c schlug Maxwell und anderen vor, dass Maxwells Gleichungen nicht in allen Bezugsrahmen wahr sind. Stattdessen, so dachten sie, seien Maxwells Gleichungen nur in einem System, dem Ruhesystem des Äthers, exakt wahr, und in allen anderen Systemen müssten sie durch andere Gleichungen ersetzt werden, Gleichungen, die unter Galilei-Transformationen unveränderlich seien, um dem Prinzip zu entsprechen der Relativität. Diese anderen Gleichungen implizierten, dass die Lichtgeschwindigkeit in anderen Frames tatsächlich war c + v oder c v , wo v ist die Geschwindigkeit des Äthers. Aber dann das Michelson-Morley-Experiment, das die Geschwindigkeit finden sollte v des Äthers, zeigte schließlich, dass die Lichtgeschwindigkeit war c in allen Rahmen, was anscheinend dem Relativitätsprinzip widerspricht. Aber Einstein hat gezeigt, dass dies dem Relativitätsprinzip überhaupt nicht widerspricht, man muss nur seine Vorstellungen von Raum und Zeit überdenken.

Aber meine Frage ist, was sind die Gleichungen, von denen die Leute dachten, dass sie in anderen Rahmen als dem Ätherrahmen wahr sind? Anders ausgedrückt, was sind die Gleichungen, die man erhält, wenn man eine Galilei-Transformation auf die Maxwell-Gleichungen anwendet? (Im Gegensatz zu einer Lorentz-Transformation, die die Maxwell-Gleichungen unverändert lässt.)

Ich habe die erhaltenen Gleichungen tatsächlich schon einmal gesehen. Sie wurden von einem Physiker des 19. Jahrhunderts formuliert, vielleicht Hertz oder Heaviside, und beinhalten das Hinzufügen geschwindigkeitsabhängiger Terme zum Ampere-Maxwell-Gesetz und zum Faradayschen Gesetz. (Das heißt, abhängig von der Geschwindigkeit des Äthers.) Aber ich erinnere mich nicht an die Details.

Die Maxwell-Gleichungen haben zwei verschiedene Galileische Grenzen, eine elektrische Grenze und eine magnetische Grenze. Siehe Marc De Montigny, Germain Rousseaux, „On the electrodynamics of moving bodys at low velocities“, arxiv.org/abs/physics/0512200 . Auch physical.stackexchange.com/q/30999 , arxiv.org/abs/1112.1466 , arxiv.org/abs/physics/0606228

Antworten (1)

Ich bin kein Experte für die historische Entwicklung des Themas, aber ich werde eine Herleitung anbieten.

Betrachten Sie zwei Referenzrahmen S und S ' , und nehme das an S ' bewegt sich mit Geschwindigkeit v in Gedenken an S . Koordinaten ein S und S ' sind durch eine Galilei-Transformation verwandt:

{ t ' = t x ' = x v t
Um herauszufinden, wie sich die Felder transformieren, stellen wir fest, dass sich eine Lorentz-Transformation im Grenzwert auf eine Galilei-Transformation reduziert c . Tatsächlich transformieren sich die Felder unter einer Lorentz-Transformation wie folgt :
{ E ' = γ ( E + v × B ) ( γ 1 ) ( E v ^ ) v ^ B ' = γ ( B 1 c 2 v × E ) ( γ 1 ) ( B v ^ ) v ^
Die Grenze nehmen c damit γ 1 , erhalten wir die Galileischen Transformationen der Körper:
{ E ' = E + v × B B ' = B
Wir können dann die Transformation durch Senden rückgängig machen v v :
{ E = E ' v × B ' B = B '
Aus der gleichen Überlegung kann man die galiläische Transformation der Quellen erhalten:
{ J = J ' + ρ ' v ρ = ρ '
Wir wissen, dass die Felder und Quellen die Maxwell-Gleichungen in erfüllen S :
{ E = ρ / ϵ 0 B = 0 × E = B t × B = μ 0 ( J + ϵ 0 E t )
Ersetzen der Felder und Quellen in S mit denen drin S ' wir erhalten:
{ ( E ' v × B ' ) = ρ ' / ϵ 0 B ' = 0 × ( E ' v × B ' ) = B ' t × B ' = μ 0 ( J ' + ρ ' v + ϵ 0 ( E ' v × B ' ) t )
Als letzten Schritt müssen wir Derivate in ersetzen S mit Derivaten in S ' . Wir haben:
{ = ' t = t ' v
Durch Ersetzen und Entfernen der Primzahlen und Verwendung der Vektorrechnung erhalten wir:
{ E + v ( × B ) = ρ / ϵ 0 B = 0 × E = B t × B = μ 0 ( J + ρ v + ϵ 0 t ( E v × B ) ϵ 0 v ( E v × B ) )

In einem Vakuum können wir die Lockung der vierten Gleichung nehmen, um zu erhalten:

c 2 2 B = 2 B t 2 + ( v ) 2 B 2 v ( B t )
Ersetzen einer Wellenlösung der Form B exp ich ( k x ω t ) Wir erhalten eine Gleichung für ω , die wir lösen können, um zu erhalten:
ω = v k ± c | k |
Daher ist die Ausbreitungsgeschwindigkeit die Gruppengeschwindigkeit:
ω k = v ± c  k ^
was Ihnen das erwartete gibt c ± v mit entsprechender Auswahl v und k .

wow, sehr beeindruckend, +1. -NN